Happy Memorial Day! Please note that AoPS Online is closed May 24-26th.

G
Topic
First Poster
Last Poster
k a May Highlights and 2025 AoPS Online Class Information
jlacosta   0
May 1, 2025
May is an exciting month! National MATHCOUNTS is the second week of May in Washington D.C. and our Founder, Richard Rusczyk will be presenting a seminar, Preparing Strong Math Students for College and Careers, on May 11th.

Are you interested in working towards MATHCOUNTS and don’t know where to start? We have you covered! If you have taken Prealgebra, then you are ready for MATHCOUNTS/AMC 8 Basics. Already aiming for State or National MATHCOUNTS and harder AMC 8 problems? Then our MATHCOUNTS/AMC 8 Advanced course is for you.

Summer camps are starting next month at the Virtual Campus in math and language arts that are 2 - to 4 - weeks in duration. Spaces are still available - don’t miss your chance to have an enriching summer experience. There are middle and high school competition math camps as well as Math Beasts camps that review key topics coupled with fun explorations covering areas such as graph theory (Math Beasts Camp 6), cryptography (Math Beasts Camp 7-8), and topology (Math Beasts Camp 8-9)!

Be sure to mark your calendars for the following upcoming events:
[list][*]May 9th, 4:30pm PT/7:30pm ET, Casework 2: Overwhelming Evidence — A Text Adventure, a game where participants will work together to navigate the map, solve puzzles, and win! All are welcome.
[*]May 19th, 4:30pm PT/7:30pm ET, What's Next After Beast Academy?, designed for students finishing Beast Academy and ready for Prealgebra 1.
[*]May 20th, 4:00pm PT/7:00pm ET, Mathcamp 2025 Qualifying Quiz Part 1 Math Jam, Problems 1 to 4, join the Canada/USA Mathcamp staff for this exciting Math Jam, where they discuss solutions to Problems 1 to 4 of the 2025 Mathcamp Qualifying Quiz!
[*]May 21st, 4:00pm PT/7:00pm ET, Mathcamp 2025 Qualifying Quiz Part 2 Math Jam, Problems 5 and 6, Canada/USA Mathcamp staff will discuss solutions to Problems 5 and 6 of the 2025 Mathcamp Qualifying Quiz![/list]
Our full course list for upcoming classes is below:
All classes run 7:30pm-8:45pm ET/4:30pm - 5:45pm PT unless otherwise noted.

Introductory: Grades 5-10

Prealgebra 1 Self-Paced

Prealgebra 1
Tuesday, May 13 - Aug 26
Thursday, May 29 - Sep 11
Sunday, Jun 15 - Oct 12
Monday, Jun 30 - Oct 20
Wednesday, Jul 16 - Oct 29

Prealgebra 2 Self-Paced

Prealgebra 2
Wednesday, May 7 - Aug 20
Monday, Jun 2 - Sep 22
Sunday, Jun 29 - Oct 26
Friday, Jul 25 - Nov 21

Introduction to Algebra A Self-Paced

Introduction to Algebra A
Sunday, May 11 - Sep 14 (1:00 - 2:30 pm ET/10:00 - 11:30 am PT)
Wednesday, May 14 - Aug 27
Friday, May 30 - Sep 26
Monday, Jun 2 - Sep 22
Sunday, Jun 15 - Oct 12
Thursday, Jun 26 - Oct 9
Tuesday, Jul 15 - Oct 28

Introduction to Counting & Probability Self-Paced

Introduction to Counting & Probability
Thursday, May 15 - Jul 31
Sunday, Jun 1 - Aug 24
Thursday, Jun 12 - Aug 28
Wednesday, Jul 9 - Sep 24
Sunday, Jul 27 - Oct 19

Introduction to Number Theory
Friday, May 9 - Aug 1
Wednesday, May 21 - Aug 6
Monday, Jun 9 - Aug 25
Sunday, Jun 15 - Sep 14
Tuesday, Jul 15 - Sep 30

Introduction to Algebra B Self-Paced

Introduction to Algebra B
Tuesday, May 6 - Aug 19
Wednesday, Jun 4 - Sep 17
Sunday, Jun 22 - Oct 19
Friday, Jul 18 - Nov 14

Introduction to Geometry
Sunday, May 11 - Nov 9
Tuesday, May 20 - Oct 28
Monday, Jun 16 - Dec 8
Friday, Jun 20 - Jan 9
Sunday, Jun 29 - Jan 11
Monday, Jul 14 - Jan 19

Paradoxes and Infinity
Mon, Tue, Wed, & Thurs, Jul 14 - Jul 16 (meets every day of the week!)

Intermediate: Grades 8-12

Intermediate Algebra
Sunday, Jun 1 - Nov 23
Tuesday, Jun 10 - Nov 18
Wednesday, Jun 25 - Dec 10
Sunday, Jul 13 - Jan 18
Thursday, Jul 24 - Jan 22

Intermediate Counting & Probability
Wednesday, May 21 - Sep 17
Sunday, Jun 22 - Nov 2

Intermediate Number Theory
Sunday, Jun 1 - Aug 24
Wednesday, Jun 18 - Sep 3

Precalculus
Friday, May 16 - Oct 24
Sunday, Jun 1 - Nov 9
Monday, Jun 30 - Dec 8

Advanced: Grades 9-12

Olympiad Geometry
Tuesday, Jun 10 - Aug 26

Calculus
Tuesday, May 27 - Nov 11
Wednesday, Jun 25 - Dec 17

Group Theory
Thursday, Jun 12 - Sep 11

Contest Preparation: Grades 6-12

MATHCOUNTS/AMC 8 Basics
Friday, May 23 - Aug 15
Monday, Jun 2 - Aug 18
Thursday, Jun 12 - Aug 28
Sunday, Jun 22 - Sep 21
Tues & Thurs, Jul 8 - Aug 14 (meets twice a week!)

MATHCOUNTS/AMC 8 Advanced
Sunday, May 11 - Aug 10
Tuesday, May 27 - Aug 12
Wednesday, Jun 11 - Aug 27
Sunday, Jun 22 - Sep 21
Tues & Thurs, Jul 8 - Aug 14 (meets twice a week!)

AMC 10 Problem Series
Friday, May 9 - Aug 1
Sunday, Jun 1 - Aug 24
Thursday, Jun 12 - Aug 28
Tuesday, Jun 17 - Sep 2
Sunday, Jun 22 - Sep 21 (1:00 - 2:30 pm ET/10:00 - 11:30 am PT)
Monday, Jun 23 - Sep 15
Tues & Thurs, Jul 8 - Aug 14 (meets twice a week!)

AMC 10 Final Fives
Sunday, May 11 - Jun 8
Tuesday, May 27 - Jun 17
Monday, Jun 30 - Jul 21

AMC 12 Problem Series
Tuesday, May 27 - Aug 12
Thursday, Jun 12 - Aug 28
Sunday, Jun 22 - Sep 21
Wednesday, Aug 6 - Oct 22

AMC 12 Final Fives
Sunday, May 18 - Jun 15

AIME Problem Series A
Thursday, May 22 - Jul 31

AIME Problem Series B
Sunday, Jun 22 - Sep 21

F=ma Problem Series
Wednesday, Jun 11 - Aug 27

WOOT Programs
Visit the pages linked for full schedule details for each of these programs!


MathWOOT Level 1
MathWOOT Level 2
ChemWOOT
CodeWOOT
PhysicsWOOT

Programming

Introduction to Programming with Python
Thursday, May 22 - Aug 7
Sunday, Jun 15 - Sep 14 (1:00 - 2:30 pm ET/10:00 - 11:30 am PT)
Tuesday, Jun 17 - Sep 2
Monday, Jun 30 - Sep 22

Intermediate Programming with Python
Sunday, Jun 1 - Aug 24
Monday, Jun 30 - Sep 22

USACO Bronze Problem Series
Tuesday, May 13 - Jul 29
Sunday, Jun 22 - Sep 1

Physics

Introduction to Physics
Wednesday, May 21 - Aug 6
Sunday, Jun 15 - Sep 14
Monday, Jun 23 - Sep 15

Physics 1: Mechanics
Thursday, May 22 - Oct 30
Monday, Jun 23 - Dec 15

Relativity
Mon, Tue, Wed & Thurs, Jun 23 - Jun 26 (meets every day of the week!)
0 replies
jlacosta
May 1, 2025
0 replies
k i Adding contests to the Contest Collections
dcouchman   1
N Apr 5, 2023 by v_Enhance
Want to help AoPS remain a valuable Olympiad resource? Help us add contests to AoPS's Contest Collections.

Find instructions and a list of contests to add here: https://artofproblemsolving.com/community/c40244h1064480_contests_to_add
1 reply
dcouchman
Sep 9, 2019
v_Enhance
Apr 5, 2023
k i Zero tolerance
ZetaX   49
N May 4, 2019 by NoDealsHere
Source: Use your common sense! (enough is enough)
Some users don't want to learn, some other simply ignore advises.
But please follow the following guideline:


To make it short: ALWAYS USE YOUR COMMON SENSE IF POSTING!
If you don't have common sense, don't post.


More specifically:

For new threads:


a) Good, meaningful title:
The title has to say what the problem is about in best way possible.
If that title occured already, it's definitely bad. And contest names aren't good either.
That's in fact a requirement for being able to search old problems.

Examples:
Bad titles:
- "Hard"/"Medium"/"Easy" (if you find it so cool how hard/easy it is, tell it in the post and use a title that tells us the problem)
- "Number Theory" (hey guy, guess why this forum's named that way¿ and is it the only such problem on earth¿)
- "Fibonacci" (there are millions of Fibonacci problems out there, all posted and named the same...)
- "Chinese TST 2003" (does this say anything about the problem¿)
Good titles:
- "On divisors of a³+2b³+4c³-6abc"
- "Number of solutions to x²+y²=6z²"
- "Fibonacci numbers are never squares"


b) Use search function:
Before posting a "new" problem spend at least two, better five, minutes to look if this problem was posted before. If it was, don't repost it. If you have anything important to say on topic, post it in one of the older threads.
If the thread is locked cause of this, use search function.

Update (by Amir Hossein). The best way to search for two keywords in AoPS is to input
[code]+"first keyword" +"second keyword"[/code]
so that any post containing both strings "first word" and "second form".


c) Good problem statement:
Some recent really bad post was:
[quote]$lim_{n\to 1}^{+\infty}\frac{1}{n}-lnn$[/quote]
It contains no question and no answer.
If you do this, too, you are on the best way to get your thread deleted. Write everything clearly, define where your variables come from (and define the "natural" numbers if used). Additionally read your post at least twice before submitting. After you sent it, read it again and use the Edit-Button if necessary to correct errors.


For answers to already existing threads:


d) Of any interest and with content:
Don't post things that are more trivial than completely obvious. For example, if the question is to solve $x^{3}+y^{3}=z^{3}$, do not answer with "$x=y=z=0$ is a solution" only. Either you post any kind of proof or at least something unexpected (like "$x=1337, y=481, z=42$ is the smallest solution). Someone that does not see that $x=y=z=0$ is a solution of the above without your post is completely wrong here, this is an IMO-level forum.
Similar, posting "I have solved this problem" but not posting anything else is not welcome; it even looks that you just want to show off what a genius you are.

e) Well written and checked answers:
Like c) for new threads, check your solutions at least twice for mistakes. And after sending, read it again and use the Edit-Button if necessary to correct errors.



To repeat it: ALWAYS USE YOUR COMMON SENSE IF POSTING!


Everything definitely out of range of common sense will be locked or deleted (exept for new users having less than about 42 posts, they are newbies and need/get some time to learn).

The above rules will be applied from next monday (5. march of 2007).
Feel free to discuss on this here.
49 replies
ZetaX
Feb 27, 2007
NoDealsHere
May 4, 2019
Hardest in ARO 2008
discredit   26
N a few seconds ago by JARP091
Source: ARO 2008, Problem 11.8
In a chess tournament $ 2n+3$ players take part. Every two play exactly one match. The schedule is such that no two matches are played at the same time, and each player, after taking part in a match, is free in at least $ n$ next (consecutive) matches. Prove that one of the players who play in the opening match will also play in the closing match.
26 replies
discredit
Jun 11, 2008
JARP091
a few seconds ago
Inequality
Kei0923   2
N 28 minutes ago by Kei0923
Source: Own.
Let $k\leq 1$ be a fixed positive real number. Find the minimum possible value $M$ such that for any positive reals $a$, $b$, $c$, $d$, we have
$$\sqrt{\frac{ab}{(a+b)(b+c)}}+\sqrt{\frac{cd}{(c+d)(d+ka)}}\leq M.$$
2 replies
Kei0923
Jul 25, 2023
Kei0923
28 minutes ago
PAMO 2023 Problem 2
kerryberry   6
N 34 minutes ago by justaguy_69
Source: 2023 Pan African Mathematics Olympiad Problem 2
Find all positive integers $m$ and $n$ with no common divisor greater than 1 such that $m^3 + n^3$ divides $m^2 + 20mn + n^2$. (Professor Yongjin Song)
6 replies
kerryberry
May 17, 2023
justaguy_69
34 minutes ago
My Unsolved Problem
ZeltaQN2008   0
an hour ago
Source: IDK
Given a positive integer \( m \) and \( a > 1 \). Prove that there always exists a positive integer \( n \) such that \( m \mid (a^n + n) \).

P/s: I can prove the problem if $m$ is a power of a prime number, but for arbitrary $m$ then well.....
0 replies
ZeltaQN2008
an hour ago
0 replies
Computing functions
BBNoDollar   3
N an hour ago by wh0nix
Let $f : [0, \infty) \to [0, \infty)$, $f(x) = \dfrac{ax + b}{cx + d}$, with $a, d \in (0, \infty)$, $b, c \in [0, \infty)$. Prove that there exists $n \in \mathbb{N}^*$ such that for every $x \geq 0$
\[
f_n(x) = \frac{x}{1 + nx}, \quad \text{if and only if } f(x) = \frac{x}{1 + x}, \quad \forall x \geq 0.
\](For $n \in \mathbb{N}^*$ and $x \geq 0$, the notation $f_n(x)$ represents $\underbrace{(f \circ f \circ \dots \circ f)}_{n \text{ times}}(x)$. )
3 replies
BBNoDollar
May 21, 2025
wh0nix
an hour ago
Computing functions
BBNoDollar   8
N an hour ago by wh0nix
Let $f : [0, \infty) \to [0, \infty)$, $f(x) = \dfrac{ax + b}{cx + d}$, with $a, d \in (0, \infty)$, $b, c \in [0, \infty)$. Prove that there exists $n \in \mathbb{N}^*$ such that for every $x \geq 0$
\[
f_n(x) = \frac{x}{1 + nx}, \quad \text{if and only if } f(x) = \frac{x}{1 + x}, \quad \forall x \geq 0.
\](For $n \in \mathbb{N}^*$ and $x \geq 0$, the notation $f_n(x)$ represents $\underbrace{(f \circ f \circ \dots \circ f)}_{n \text{ times}}(x)$. )
8 replies
BBNoDollar
May 18, 2025
wh0nix
an hour ago
Find the remainder
Jackson0423   1
N an hour ago by wh0nix

Find the remainder when
\[
\frac{5^{2000} - 1}{4}
\]is divided by \(64\).
1 reply
Jackson0423
2 hours ago
wh0nix
an hour ago
IMO 2018 Problem 1
juckter   170
N an hour ago by Adywastaken
Let $\Gamma$ be the circumcircle of acute triangle $ABC$. Points $D$ and $E$ are on segments $AB$ and $AC$ respectively such that $AD = AE$. The perpendicular bisectors of $BD$ and $CE$ intersect minor arcs $AB$ and $AC$ of $\Gamma$ at points $F$ and $G$ respectively. Prove that lines $DE$ and $FG$ are either parallel or they are the same line.

Proposed by Silouanos Brazitikos, Evangelos Psychas and Michael Sarantis, Greece
170 replies
juckter
Jul 9, 2018
Adywastaken
an hour ago
Nice "if and only if" function problem
ICE_CNME_4   2
N an hour ago by wh0nix
Let $f : [0, \infty) \to [0, \infty)$, $f(x) = \dfrac{ax + b}{cx + d}$, with $a, d \in (0, \infty)$, $b, c \in [0, \infty)$. Prove that there exists $n \in \mathbb{N}^*$ such that for every $x \geq 0$
\[
f_n(x) = \frac{x}{1 + nx}, \quad \text{if and only if } f(x) = \frac{x}{1 + x}, \quad \forall x \geq 0.
\](For $n \in \mathbb{N}^*$ and $x \geq 0$, the notation $f_n(x)$ represents $\underbrace{(f \circ f \circ \dots \circ f)}_{n \text{ times}}(x)$. )

Please do it at 9th grade level. Thank you!
2 replies
ICE_CNME_4
Yesterday at 7:23 PM
wh0nix
an hour ago
Minimum of this fuction
persamaankuadrat   1
N 2 hours ago by alexheinis
Source: KTOM January 2020
If $x$ is a positive real number, find the minimum of the following expression

$$\lfloor x \rfloor + \frac{500}{\lceil x\rceil^{2}}$$
1 reply
persamaankuadrat
3 hours ago
alexheinis
2 hours ago
A well-known circle hides in the dark
darij grinberg   15
N 3 hours ago by TUAN2k8
Source: All-Russian Olympiad 2006 finals, problem 11.4 (problem 4 for grade 11)
Given a triangle $ ABC$. The angle bisectors of the angles $ ABC$ and $ BCA$ intersect the sides $ CA$ and $ AB$ at the points $ B_1$ and $ C_1$, and intersect each other at the point $ I$. The line $ B_1C_1$ intersects the circumcircle of triangle $ ABC$ at the points $ M$ and $ N$. Prove that the circumradius of triangle $ MIN$ is twice as long as the circumradius of triangle $ ABC$.
15 replies
1 viewing
darij grinberg
May 6, 2006
TUAN2k8
3 hours ago
unsolved nt
MuradSafarli   1
N 3 hours ago by whwlqkd
Find all prime numbers $p$ such that $p^2 - 12$ is also a prime number.
1 reply
MuradSafarli
3 hours ago
whwlqkd
3 hours ago
IMO Problem 4
iandrei   106
N 3 hours ago by alexanderchew
Source: IMO ShortList 2003, geometry problem 1
Let $ABCD$ be a cyclic quadrilateral. Let $P$, $Q$, $R$ be the feet of the perpendiculars from $D$ to the lines $BC$, $CA$, $AB$, respectively. Show that $PQ=QR$ if and only if the bisectors of $\angle ABC$ and $\angle ADC$ are concurrent with $AC$.
106 replies
iandrei
Jul 14, 2003
alexanderchew
3 hours ago
interesting diophantiic fe in natural numbers
skellyrah   0
3 hours ago
Find all functions \( f : \mathbb{N} \to \mathbb{N} \) such that for all \( m, n \in \mathbb{N} \),
\[
mn + f(n!) = f(f(n))! + n \cdot \gcd(f(m), m!).
\]
0 replies
skellyrah
3 hours ago
0 replies
number of divisors
orl   27
N Apr 27, 2025 by Maximilian113
Source: IMO Shortlist 2000, Problem N2
For a positive integer $n$, let $d(n)$ be the number of all positive divisors of $n$. Find all positive integers $n$ such that $d(n)^3=4n$.
27 replies
orl
Sep 6, 2003
Maximilian113
Apr 27, 2025
number of divisors
G H J
G H BBookmark kLocked kLocked NReply
Source: IMO Shortlist 2000, Problem N2
The post below has been deleted. Click to close.
This post has been deleted. Click here to see post.
orl
3647 posts
#1 • 4 Y
Y by Adventure10, centslordm, Mango247, MarioLuigi8972
For a positive integer $n$, let $d(n)$ be the number of all positive divisors of $n$. Find all positive integers $n$ such that $d(n)^3=4n$.
This post has been edited 2 times. Last edited by djmathman, Jul 22, 2016, 11:16 PM
Z K Y
The post below has been deleted. Click to close.
This post has been deleted. Click here to see post.
hitlEULER
45 posts
#2 • 3 Y
Y by centslordm, Adventure10, Adventure10
Of course $n > 1$. So let $n = \prod_{i=1}^r p_i^{a_i}$, where $r$ is a positive integer, $p_1 < p_2 < \ldots < p_r$ are pairwise distinct natural primes and $a_i \in \mathbb{Z}^+$, for any $i = 1, 2, \ldots, r$. We need $\prod_{i=1}^r (1 + a_i)^3 = 4\prod_{i=1}^r p_i^{a_i}$, and $v_2(\mbox{LHS}) = v_2(\mbox{RHS}) > 0$, where $v_2(\cdot)$ is the $2$-adic valuation of its argument.

Well, actually $3 \mid v_2(\mbox{LHS})$, so a fortiori $p_1 = 2$ and $a_1 = 3b_1 + 1$, for a certain $b_1 \in \mathbb{N}$. Moreover similarly $a_i = 3b_i$, where $b_i \in \mathbb{Z}^+$, if $i = 2, 3, \ldots, r$. If $r = 1$, this leads uniquely to $2+3b_1 = 2^{b_1+1}$ and $b_1 = 1$ or $b_1 = 2$, viz $n = 2$ or $n = 2^7$. Hence let $r > 1$. You get $(2+3b_1)\prod_{i=2}^r (1+3b_i) = 2^{b_1 + 1} \prod_{i=2}^r p_i^{b_i}$. So necessarily $p_2 \geq 5$, as $\mbox{LHS} \equiv 2 \bmod 3$.

If $b_1 = 1$, this means $5\cdot \prod_{i=2}^r (1+3b_i) = 2^2 \prod_{i=2}^r p_i^{b_i}$, leading to $p_2 = 5$, $b_2 = 1$ and $\prod_{i=3}^r (1 + 3b_i) = \prod_{i=3}^r p_i^{b_i}$, which is possible iff both the products in the left and right sides are empty, since by Bernoulli's inequality: $p^a > (1+3)^a \geq (1+3a)$, for any $a \in \mathbb{N}$ and any real $p > 4$. The same argument proves there's no further solution for $b_1 \geq 2$, because that implies $2+3b_1 \leq 2^{b_1 + 1}$, by induction. Then $d^3(n) = 4n$ iff $n = 2$, $n = 2^7$ or $n = 2^4 \cdot 5^3$.
Z K Y
The post below has been deleted. Click to close.
This post has been deleted. Click here to see post.
abdurashidjon
119 posts
#3 • 2 Y
Y by centslordm, Adventure10
hitlEULER wrote:
Of course $n > 1$. So let $n = \prod_{i=1}^r p_i^{a_i}$, where $r$ is a positive integer, $p_1 < p_2 < \ldots < p_r$ are pairwise distinct natural primes and $a_i \in \mathbb{Z}^+$, for any $i = 1, 2, \ldots, r$. We need $\prod_{i=1}^r (1 + a_i)^3 = 4\prod_{i=1}^r p_i^{a_i}$, and $v_2(\mbox{LHS}) = v_2(\mbox{RHS}) > 0$, where $v_2(\cdot)$ is the $2$-adic valuation of its argument.

Well, actually $3 \mid v_2(\mbox{LHS})$, so a fortiori $p_1 = 2$ and $a_1 = 3b_1 + 1$, for a certain $b_1 \in \mathbb{N}$. Moreover similarly $a_i = 3b_i$, where $b_i \in \mathbb{Z}^+$, if $i = 2, 3, \ldots, r$. If $r = 1$, this leads uniquely to $2+3b_1 = 2^{b_1+1}$ and $b_1 = 1$ or $b_1 = 2$, viz $n = 2$ or $n = 2^7$. Hence let $r > 1$. You get $(2+3b_1)\prod_{i=2}^r (1+3b_i) = 2^{b_1 + 1} \prod_{i=2}^r p_i^{b_i}$. So necessarily $p_2 \geq 5$, as $\mbox{LHS} \equiv 2 \bmod 3$.

If $b_1 = 1$, this means $5\cdot \prod_{i=2}^r (1+3b_i) = 2^2 \prod_{i=2}^r p_i^{b_i}$, leading to $p_2 = 5$, $b_2 = 1$ and $\prod_{i=3}^r (1 + 3b_i) = \prod_{i=3}^r p_i^{b_i}$, which is possible iff both the products in the left and right sides are empty, since by Bernoulli's inequality: $p^a > (1+3)^a \geq (1+3a)$, for any $a \in \mathbb{N}$ and any real $p > 4$. The same argument proves there's no further solution for $b_1 \geq 2$, because that implies $2+3b_1 \leq 2^{b_1 + 1}$, by induction. Then $d^3(n) = 4n$ iff $n = 2$, $n = 2^7$ or $n = 2^4 \cdot 5^3$.

Here you have used fortori 2-adic what does these words mean
Abdurashid
Z K Y
The post below has been deleted. Click to close.
This post has been deleted. Click here to see post.
Omid Hatami
1275 posts
#4 • 3 Y
Y by centslordm, Adventure10, Seungjun_Lee
Well $p$-adic numbers are completion of $\mathbb Q$ with the metric $\rho(x,y)=\parallel x-y\parallel _{p}$ which if $x=\frac{a}{b}$ which $(a,b)=1$ then if $p|a$ $\parallel x\parallel _{p}=\parallel a\parallel _{p}$ if not then$\parallel x\parallel _{p}=-\parallel b\parallel _{p}$. You could see they are like infinite digit numbers $\overline{\dots a_{2}a_{1}a_{0}.b_{1}b_{2}\dots b_{k}}$
Z K Y
The post below has been deleted. Click to close.
This post has been deleted. Click here to see post.
va2010
1276 posts
#5 • 3 Y
Y by centslordm, Adventure10, Mango247
The intended solution is probably like the following. Let $f(x) = \frac{d(x)^3}{x}$. Observe that $f(x)$ is multiplicative. Also, $n$ is two times a perfect cube. Hence, $v_2(n) \equiv 1 \pmod{3}$ and $v_p(n) \equiv 0 \pmod{3}$ for odd primes $p$.

Lemma: $v_3(n) = 0$

Proof: Assume otherwise, then using multiplicativity of $f(x)$, an exponent of $3$ gets introduced to the denominator, and in fact it is not possible for this to be eliminated. $\blacksquare$.

Now we can determine through tedious casework that $f(x) < 1$ if $x$ is a prime power of a prime greater than $3$. Hence, the only way for a prime beyond this threshold to work is if $f(2^k)$ makes up for it. We can determine that $f(2^7) = 4$ and $f(2) = 4$, and $f(2^{3k+1}) < 4$ for $k > 2$. For $k=1$, the only other possibility is multiplication by $5^3$. Hence the solutions are $n = 2^7$, $n = 2$, $n = 2^4 * 5^3$, which all work.

Comments: For similar problems, observe HMIC 2015.1, Spring OMO 2014.19, and Canada 1999.3
Z K Y
The post below has been deleted. Click to close.
This post has been deleted. Click here to see post.
Wave-Particle
3690 posts
#6 • 3 Y
Y by centslordm, Adventure10, Mango247
I believe the problem should read as follows,

"For a positive integer $n$, let $d(n)$ be the number of all positive divisors of $n$. Find all positive integers $n$ such that $d(n)^3=4n$."

I would appreciate if a moderator of this forum or site admin could edit the OP to match it so it'll read correctly on the contests section as well :)
Z K Y
The post below has been deleted. Click to close.
This post has been deleted. Click here to see post.
efang
593 posts
#7 • 4 Y
Y by samuel, centslordm, Adventure10, EquationTracker
p-adic evaluations are not necessary for this problem; you can solve it with some pretty silly bounds

Clearly, the prime factorization of $n$ must be of the form $n = 2^{3x-2}*\prod_{i=1}^r p_i^{3e_i}$.

Now from $d(n) = \sqrt[3]{4n}$ we have $(3x-1)*\prod_{i=1}(3e_i+1) = 2^x*\prod_{i=1}^r p_i^{e_i}$

First, the LHS of this equation clearly has no factors of 3 and thus none of the $p_i$ of the RHS can be 3. (1)

Second, $\forall p_i > 4$ we must have $p_i^{e_i} > 3e_i+1$ (easy to prove with induction) (2)

Third, for $x > 3$ we have $2^x > 3x-1$

Thus, we must have $x \le 3$.

Case 1: $x=1$

Then, we have $2^x = 3x-1$. Thus, from (2) we conclude that there can be no other prime factors in this case. Thus we have $n = 2^{3x-2} = 2$

Case 2: $x=2$

Then, we have $2^x < 3x-1$. This makes it possible to have other prime factors. Notice that $3x-1 = 5$ thus 5 must be one of the prime factors. So we let $p_1 = 5$. Now, if $e_1 > 1$ then we have that $2^2*5^{e_1} > 5(3e_1+1)$. Thus, concluding from (2) again we see that $e_1 = 1$

When $e_1 = 1$ we have equality. Thus, by observation (2) again n cannot have any additional prime factors and this must be the only solution in this case.

The solution is $n = 2^{3x-2}*5^{3e_1} = 2^4*5^3$

Case 3: $x=3$

In this case we have $2^x = 3x-1$ and again like in case 1 we may conclude that n has no other prime factors. Thus the only solution in this case is $n = 2^{3x-2} = 2^7$

Thus, our solutions are $\boxed{2, 2^4*5^3, 2^7}$
Z K Y
The post below has been deleted. Click to close.
This post has been deleted. Click here to see post.
v_Enhance
6877 posts
#8 • 7 Y
Y by UzbekMathematician, Not_real_name, Mathasocean, v4913, centslordm, Adventure10, Mango247
The answer is $n=2$, $n=128$, and $n = 2000$. These are checked to work.

In general, we can let \[ n = 2^{3r+1} \cdot p_1^{3e_1} \dots p_k^{3e_k} \]where $p_i$ are distinct odd primes, and $e_i > 0$. We assume $n > 1$ throughout (as $n=1$ fails). Then $d(n)^3 = 4n$ rearranges as \[ 2^{r+1} p_1^{e_1} \dots p_k^{e_k} 	= (3r+2)(3e_1+1) \dots (3e_k+1). \]We now split into several cases.
  • Suppose $r = 0$ or $r = 2$, so the equation becomes \[ p_1^{e_1} \dots p_k^{e_k} = (3e_1+1) \dots (3e_k+1). \]Then $e_i \neq 1$ for all $i$, since the right-hand side is odd.

    But then $p_i^{e_i} \ge 3^{e_i} > 3e_i + 1$ for $e_i \ge 2$. Multiplying all these gives a contradiction unless $k = 0$. These two cases give $n = 2 \cdot 1^3 = 2$ and $n = 2 \cdot 4^3 = 128$ respectively.
  • Suppose $r = 1$, so we have \[ 4p_1^{e_1} \dots p_k^{e_k} = 5(3e_1+1) \dots (3e_k+1). \]Then $5$ is among the $p_i$, so let us assume $p_1 = 5$. We have two cases:
    • Suppose $e_1 = 1$. Then the equation becomes $p_2^{e_2} \dots p_k^{e_k} = (3e_2+1) \dots (3e_k+1)$. Then and as in the first case, we get $e_i \neq 1$ for any $i \ge 2$, and thus there can be no other primes. This gives the solution $n = 2 \cdot 10^3 = 2000$.
    • Now suppose $e_1 \ge 2$. Then we multiply the estimates \begin{align*} 			5^{e_1} &\ge \tfrac{25}{7} (3e_1 + 1) \quad 				\text{since } e_i \ge 2\\ 			3^{e_i} &\ge \tfrac34(3e_i+1) \quad \text{if } p_i = 3 \\ 			p_i^{e_i} &\ge 3e_i +1 \qquad \text{if } p_i \ge 7. 		\end{align*}to get a contradiction since $\frac{25}{7} \cdot \frac{3}{4} > \frac54$.
  • Now suppose $r \ge 3$. We then multiply the estimates \begin{align*} 		2^{r+1} &\ge \tfrac{16}{11} (3r+2) \qquad r \ge 3 \\ 		3^{e_i} &\ge \tfrac34(3e_i+1) \quad \text{if } p_i = 3 \\ 		p_i^{e_i} &\ge 3e_i+1 \qquad \text{if } p_i \ge 5. 	\end{align*}This gives another contradiction since $\frac{16}{11} \cdot \frac34 > 1$.
This post has been edited 1 time. Last edited by v_Enhance, Feb 20, 2018, 6:59 PM
Z K Y
The post below has been deleted. Click to close.
This post has been deleted. Click here to see post.
ubermensch
820 posts
#9 • 3 Y
Y by centslordm, Adventure10, Mango247
A little tedious casework, but otherwise decent problem... here's the sketch of my proof.
First observe that, given $n=p_1^{\alpha_1}p_2^{\alpha_2}...p_k^{\alpha_k}$, $p_1<p_2<...<p_k$, $p_1$ must equal $2$ and $\alpha_1=3 \beta_1 +1$ for some integer $\beta_1$ - using similar logic we get $\alpha_i =3 \beta_i$ for $2 \leq i \leq k$. Thus our equation simplifies to:
$$2^{\beta_1+1}p_2^{\beta_2}p_3^{\beta_3}...p_k^{\beta_k}=(3 \beta_1+2)(3\beta_2+1)(3 \beta_3 +1)...(3 \beta_k+1)$$
Here, a really important thing to reduce unnecessary casework is noticing that $p_2 \neq 3$ (trivial because $3$ can't divide LHS)- this will now be extremely useful because after playing around with the problem a bit, we can immediately see there'd be a heavy usage of bounding in the problem.

The pure "number theoretical" part of the problem is now over, and all that's left is a little casework... first we see that $\frac{p_i^{\beta_i}}{3 \beta_i +1} > 1$ for $p_i \geq 5$ (the proof is trivial), hence we can directly analyse when $\frac{2^{\beta_1+1}}{3 \beta_1 +2} \leq 1$ (as we need $\frac{2^{\beta_1+1}p_2^{\beta_2}p_3^{\beta_3}...p_k^{\beta_k}}{(3 \beta_1+2)(3\beta_2+1)(3 \beta_3 +1)...(3 \beta_k+1)}=1$), and from this we get $\beta_1 \leq 2$. Now plugging in $\beta_1=2$ directly gives you the only solution of its type as $2^{3 \cdot 2 +1}=2^7$, $\beta=0$ again gives an only solution as $n=2$. We have no choice but to put in $\beta_1=1$ and that'll again give an only solution (with a little more calculation) as $2^{3+1}5^3=2000$, and we're done.
This post has been edited 2 times. Last edited by ubermensch, Aug 14, 2019, 4:15 PM
Z K Y
The post below has been deleted. Click to close.
This post has been deleted. Click here to see post.
AlastorMoody
2125 posts
#10 • 3 Y
Y by centslordm, Adventure10, Mango247
Solution
Z K Y
The post below has been deleted. Click to close.
This post has been deleted. Click here to see post.
EulersTurban
386 posts
#11 • 4 Y
Y by centslordm, Mango247, Mango247, Mango247
Let $\tau(n)$ denote $d(n)$
Then, let's say that $n=2t^3$, then we have that $\tau(2t^3)^3=8t^3$, that implies $\tau(2t^3)=2t$.Now let's say that $t=p_1^{\alpha_1}p_2^{\alpha_2}\dots p_k^{\alpha_k}$
We encounter two cases.The first case is when $2 \mid t$, let's say that the primes $p_1,p_2,\dots,p_k$ are all ordered in the following way $p_1 < p_2 < p_3 < \cdots p_k$.Then we have that $p_1=2$, from here we have the relation:
$$(3\alpha_1+2)\prod_{i=2}^{k} (3\alpha_i +1) = 2^{\alpha_1+1}\prod_{i=1}^{k}p_i^{\alpha_i}$$We first examine the inequality $2^{\alpha_1+1} > 3\alpha_1+2$, we see that this holds for every $\alpha_1 > 2$, so we will examine two cases.

The first case is when $\alpha_1 = 1$, then we have that:
$$5\prod_{i=2}^{k}(3\alpha_i+1) = 4\prod_{i=1}^{k}p_i^{\alpha_i}$$We will have that one prime is $5$, and since for every prime number greater than $5$ we have the following inequality:
$$p^{a} > (3a+1)$$This holds for any $a \in N$ we choose, now we have the following:
$$\prod_{i=2}^{k} (3\alpha_i+1)=4.5^{\alpha_2-1}\prod_{i=2}^{k} p_i^{\alpha_i}$$$$(3\alpha_2+1)=4.5^{\alpha_2-1}$$From here we see that $\alpha_2=1$, and from all of this we get that $n=2^4.5^3=2000$

The second case is $\alpha_1=2$, then we have that:
$$8\prod_{i=2}^{k}(3\alpha_i+1)=2^3\prod_{i=2}^{k}p_i^{\alpha_i}$$$$\prod_{i=2}^{k}(3\alpha_i+1)=\prod_{i=2}^{k}p_i^{\alpha_i}$$Because of the inequality $p^a>3a+1$, for every prime greater than $5$ we have that $p_2=3$, but this won't hold up since we get $2 \equiv 0 \; (mod \; 3)$.So $3 \nmid t$, we have that $n=2.2^6=128$

The second general case is $2 \nmid t$, but this is easy since of the inequality $p^a > 3a+1$, we get that any prime greater than $5$ doesn't divide $t$, and especially we get that $3 \nmid n$, so we have that $t=1$, and so we get that $n=2$

To summarize the only solutions are $n \in \{ 2,128,2000 \}$ . . . :D
This post has been edited 1 time. Last edited by EulersTurban, May 18, 2020, 9:49 PM
Z K Y
The post below has been deleted. Click to close.
This post has been deleted. Click here to see post.
HKIS200543
380 posts
#12 • 1 Y
Y by centslordm
The only solutions are $n = 2, 128, 2000$, which are all seen to work. Since $2n$ is a perfect cube, we may write
\[ n = 2^{3a -2} p_1^{3e_1} p_2^{3e_2} \dots p_k^{3e_k} . \]where $p_1 < p_2 < \cdots < p_k$ and $a, e_1, e_2, \dots, e_k$ are positive integers.Then the $d(n)^3 = 4n$ is equivalent to
\[ (3a - 1)^3 (3e_1+1)^3 (3e_2 + 1)^3 \dots (3e_k+1)^3 = 2^{3a} p_1^{3p_1} p_2^{3p_2} \dots p_k^{3p_k} . \]We split into cases based on the value of $a$.
  • If $a=1$, then
    \[ (3e_1 + 1)^3 (3e_2+1)^3 \dots (3e_k+1)^3 = p_1^{3p_1} p_2^{3p_2} \dots p_k^{3p_k}. \]However, it is easily checked $p_i^{3e_i} > (e_i + 1)^3$ for all $e_i \geq 1$ whenever $p_1 \geq 3$.Hence the only solution in this case is $n = 2$.
  • If $a = 2$, then
    \[ 5^3 (3e_1 + 1)^3 (3e_2 + 1)^3 \dots (3e_k + 1)^3 = 2^6 p_1^{3e_1} p_2^{3e_2} \dots p_k^{3e_k}. \]Since 3 does not divide the left hand side, but 5 does, we must have $p_1 = 5$. Clearly $n = 2^4 \cdot 5^3$ works. Now if there exist $e_i \geq 1$ with $i \geq 2$, then we may apply our work from the previous case to deduce that $d(n)^3 < 4n$. Thus $n = 16 \cdot 5^e$. Then the equation simplifies to
    \[ 125 (3e_1+1)^3 = 64 5^{3e} . \]The right hand side increases much faster than the left hand side, so there is only one solution $e=1$. (Increasing $e$ by 1 multiplies the left right hand side by 125, but the left hand side increases by a factor of at most 8.) Thus the only solution is $n = 2^4 \cdot5 ^3 = 2000 $
  • If $a = 3$, then
    \[ (3e_1 + 1)^3 (3e_2+1)^3 \dots (3e_k+1)^3 = p_1^{3p_1} p_2^{3p_2} \dots p_k^{3p_k}. \]which collapses into the case $a = 2$. Hence the only solution here is $n = 2^{7} = 128$.
  • If $a \geq 4$, then
    \[ (3a-1)^3 < 2^{3a} . \]This implies
    \[ (3e_1 + 1)^3 (3e_2 + 1)^3 \dots (3e_k + 1)^3 <  p_1^{3e_1} p_2^{3e_2} \dots p_k^{3e_k}, \]which we have already shown to be impossible.
Having exhausted all possible cases, we may conclude that these are the only solutions.
Z K Y
The post below has been deleted. Click to close.
This post has been deleted. Click here to see post.
CaptainLevi16
35 posts
#13 • 1 Y
Y by centslordm
Let $n=p_1^{\alpha_1}p_2^{\alpha_2}...p_k^{\alpha_k}$ for $p_1<p_2<...<p_k$ where $p_1=2$. As $4n$ is a perfect cube, we write $\alpha_1=3 \beta_1 -2$ and $\alpha_i =3 \beta_i$ for $2 \leq i \leq k$.
So our final equation is
$$2^{\beta_1}p_2^{\beta_2}p_3^{\beta_3}...p_k^{\beta_k}=(3 \beta_1-1)(3\beta_2+1)(3 \beta_3 +1)...(3 \beta_k+1)$$We notice that $p_2 \neq 3$ since $3$ does not divide the RHS. Now we rearrange our equation in this format:
$$\frac{2^{\beta_1}}{(3{\beta_1}-1)}=\frac{(3{\beta_2}+1)}{p_2^{\beta_2}}\cdot\frac{(3{\beta_3}+1)}{p_3^{\beta_3}}\cdots\frac{(3{\beta_k}+1)}{p_k^{\beta_k}}$$If $p_2=5$, we can see that $5^{\beta_2}=25^{(\beta_2)/2}=(1+24)^{(\beta_2)/2}\geq(1+12\beta_2)$ by Bernoulli's inequality. Thus the first fraction on RHS is $\leq1$. Similarly we can prove this for other primes. Hence the RHS is $\leq1$. Thus we obtain that $\beta_1={1,2,3}$. For each of the cases, we get $n=128,2,2000$ respectively [after some calculation]
Z K Y
The post below has been deleted. Click to close.
This post has been deleted. Click here to see post.
Maths_1729
390 posts
#14 • 1 Y
Y by centslordm
orl wrote:
For a positive integer $n$, let $d(n)$ be the number of all positive divisors of $n$. Find all positive integers $n$ such that $d(n)^3=4n$.

Let $n=\prod_{i=1}^{r} p_i^{a_i}$ then $d(n)=\tau(n)=\prod_{i=1}^{r}(a_i+1)$ now
$(\prod_{i=1}^{r}(a_i+1))^3=4*\prod_{i=1}^{r}p_i^{a_i}\implies p_i\neq 3$ when$p_k\neq 2, 3|a_k\implies a_k=3*b_k$ when$p_k=2, a_k\equiv 1\mod 3\implies a_k=3*b_k+1$
Now let for all $p_i\neq 3,b_i\geq 1$ then $(3*b_i+1)<p_i^{3*b_i}$ and if $p_k=2$ then for all $b_k>3$ or $3*b_k+1>10$ then $(3*b_k+2)^3<2^{3b_k+1}$ so let
$n=2^{3b_1+1}*\prod_{i=2}^{r}p_i^{3*b_i}$
Clearly as mentioned above $b_1<3$ is only possible..
Case 1- $b_1=0$ we have
$2^3*\prod_{i=2}^{r}(3*b_i+1)=4*2*\prod_{i=2}^{r}p_i^{3*b_i}$ so clearly in this case $n=2$ is only solution.
Case 2- $b_1=1$ then we have
$5^3*\prod_{i=2}^{r}(3*b_i+1)=4*2^4*\prod_{i=2}^{r}p_i^{3*b_i}$ so clearly there must exist $p_k=5, a_k=3$ then
$5^3*4^3*\prod_{i=3}^{r}(3*b_i+1)=4*2^4*5^3*\prod_{i=3}^{r}p_i^{3*b_i}$ which $\prod_{i=3}^{r}(3*b_i+1)=\prod_{i=3}^{r}p_i^{3*b_i}$ for $p_i>5$ which is clear contradiction so $n$ has only 1 prime divisor $2$ . Hence only solution in this case is $n=2^4*5^3$
Case 3-) $b_i=2$ then $8^3*\prod_{i=2}^{r}(3*b_i+1)=4*2^7*\prod_{i=2}^{r}p_i^{3*b_i}$ we get
$\prod_{i=2}^{r}(3*b_i+1)=\prod_{i=2}^{r}p_i^{3*b_i}$ for $p_i>3$ this is clear contradiction so $n$ has only 1 prime divisor $2$, Hence in this case only solution is $n=2^7$
Case 4- $b_i=3$ then $11^3*\prod_{i=2}^{r}(3*b_i+1)=4*2^{10}*\prod_{i=2}^{r}p_i^{3*b_i}$
But $5^3*\prod_{i=2}^{r}(3*b_i+1)< 4*2^4*\prod_{i=2}^{r}p_i^{3*b_i}$ so this case has no solution!!
Hence $n\in\{2, 2^4*5^3, 2^7\}$ $\blacksquare$
This post has been edited 4 times. Last edited by Maths_1729, Nov 20, 2020, 6:40 AM
Z K Y
The post below has been deleted. Click to close.
This post has been deleted. Click here to see post.
mathtiger6
326 posts
#15 • 3 Y
Y by centslordm, Mango247, Mango247
The answer is $\boxed{n = 2, 2^7, 2^45^3}$.

From the given equation, we can express $n$ as $$n = 2^{3\alpha + 1}p_1^{3\alpha_1}\cdots p_k^{3\alpha_k}.$$
$d(n)^3 = 4n$ turns into $$\left((3\alpha + 2)(3\alpha_1 + 1)\cdots (3\alpha_k + 1)\right)^3 = 2^{3\alpha+3} p_1^{3\alpha_1} \cdots p_k^{3\alpha_k} \qquad \qquad (1)$$or $$(3\alpha + 2)(3\alpha_1 + 1)\cdots (3\alpha_k +1) = 2^{\alpha + 1}p_1^{\alpha_1}\cdots p_k^{\alpha_k}. \qquad \qquad (2)$$
From $(1)$, note that $3 \nmid n$. In particular, $p_i \ne 3$ for any $i$.

Consider $\alpha = 0$. From $(2)$, we have $$(3\alpha_1 + 1)\cdots (3\alpha_k + 1) = p_1^{\alpha_1}\cdots p_k^{\alpha_k}.$$Dividing by the LHS gives $$1 = \frac{p_1^{\alpha_1}\cdots p_k^{\alpha_k}}{(3\alpha_1 + 1)\cdots (3\alpha_k + 1)}. \qquad \qquad (3)$$
If $k = 0$, the RHS is $1$, giving $n=2$.

If $k \geq 1$, $p_i \geq 5$, so$$\frac{p_i^{\alpha_i}}{3\alpha_i + 1} \geq \frac{5}{3(1) + 1} = \frac{5}{4}. \qquad \qquad (4)$$This is indeed the minimum value since the numerator increases faster (exponentially) than the denominator (linear) as $\alpha_i$ increases. Clearly, the RHS of $(3)$ is then greater than $1$, and thus there are no solutions for $k \geq 1$.

Now, let $\alpha = 1$ in $(2)$. This gives, after dividing, $$\frac{5}{4} =  \frac{p_1^{\alpha_1}\cdots p_k^{\alpha_k}}{(3\alpha_1 + 1)\cdots (3\alpha_k + 1)}.$$The minimum of $(4)$ only occurs at $p_i = 5, \alpha_i = 1$. Thus, the equation above is forced to be $$\frac{5}{4} = \frac{5^1}{3(1) + 1},$$giving $n = 2^45^3$.

Lastly, let $\alpha \geq 2$ in $(2)$. We find $$1 \leq \frac{2^3}{3(2) + 2}\frac{p_1^{\alpha_1}\cdots p_k^{\alpha_k}}{(3\alpha_1 + 1)\cdots (3\alpha_k + 1)} =  \frac{p_1^{\alpha_1}\cdots p_k^{\alpha_k}}{(3\alpha_1 + 1)\cdots (3\alpha_k + 1)},$$which is similar to $(3)$ with equality $k=0$, giving $n = 2^7$.

Having exhausted all cases, we are done. $\blacksquare$
This post has been edited 1 time. Last edited by mathtiger6, Nov 30, 2020, 7:50 AM
Z K Y
The post below has been deleted. Click to close.
This post has been deleted. Click here to see post.
bluelinfish
1449 posts
#16 • 2 Y
Y by RedFlame2112, centslordm
We claim that $n=2,128,2000$ are the answers. These all work, so we prove that these are the only ones. First, notice that $4n$ must be a perfect cube, so let $n=2k^3$. Now the condition changes to $d(2k^3)=2k$, and we would still like to solve for all positive integer $k$.

Suppose that $x=v_2(k)$, and let $l=\frac{k}{2^x}$. Note that $l$ is odd. This means that the equation reduces to $$d(2^{3x+1}l^3)=2\cdot 2^x\cdot l \Rightarrow (3x+2)d(l^3)=2^x\cdot l\Rightarrow \frac{d(l^3)}{l}=\frac{2^{x+1}}{3x+2}.$$Now if we let $l=p_1^{e_1}p_2^{e_2}\ldots p_q^{e_q}$, for odd primes $p_i$ and positive integer exponenets $e_i$, we get $$\prod_{i=1}^q \frac{3e_i+1}{p_i^{e_i}}=\frac{2^{x+1}}{3x+2}.$$Notice that when $e_i>1$, the expression $\frac{3e_i+1}{p_i^{e_i}}$ is always less than one, and when $e_i=1$, $p_i=3$ is the only way to make the expression greater than one. Thus the maximum value of the LHS is $\frac{4}{3}$. If $x>2$, then $RHS>\frac{4}{3}$, so we must have $x\le 2$.

When $x=0$ or $x=2$, the RHS is equal to $1$. Then either $l=1$, which works, or we must have $p_i=3, e_i=1$, causing a term in the product to be $\frac{4}{3}$. All other combinations are less than one. The only other term that does not cause the product to go below $1$ is $\frac{4}{5}$ when $p_i=5, e_i=1$, and it doesn't work, so $l=1$ is the only solution in this case.

When $x=1$, the RHS is equal to $\frac{4}{5}$. Notice that $l=5$ only creates a term of $\frac{4}{5}$ and therefore works. As every other possible product term is less than $\frac{4}{5}$ except for $\frac{4}{3}$, we must include $1$ factor of $3$ if we are to find any other solutions. The next two largest terms that don't involve the prime $3$ are $\frac{4}{5}$ from $p_i=5, e_i=1$ and $\frac{4}{7}$ for $p_i=7, e_i=1$. However, combining these terms with $\frac{4}{3}$ gives us a product less than $\frac{4}{5}$. Thus we can only include one more term, and checking the terms shows us that this is not possible.

Therefore we have three solutions:

1. When $l=1, x=0$, we get $k=1$, implying $n=2$.
2. When $l=1, x=2$ we get $k=4$, implying $n=128$.
3. When $l=5, x=2$ we get $k=20$, implying $n=2000$.

The proof is complete.

Remarks: This is the first ISL problem I've seen that includes a nonobvious use of the year number into it.

I forgot that $n$ (and therefore $k$ and therefore $l$) cannot be a multiple of $3$, it would have made the casework a lot quicker.
This post has been edited 2 times. Last edited by bluelinfish, Jul 2, 2021, 7:41 PM
Z K Y
The post below has been deleted. Click to close.
This post has been deleted. Click here to see post.
oVlad
1746 posts
#17
Y by
To begin with, observe that $n=2k^3.$ Hence, our equation rewrites as $2k=\tau(2k^3).$

Let $k=2^a\cdot 3^b\cdot 5^c\cdot d$ and assume that $d\neq 1.$ Now, observe that for all $p\geq 7$ and $m\geq 1,$ $p^m\geq 7(3m+1)/4.$

Moreover, since $f(x)=2^{x+1}/(3x+2)$ and $g_i(x)=i^x/(3x+1), \ i\in\{3,5\}$ reach their minimums in $x=1,1$ and $0$ respectively, we have \[\frac{2k}{\tau(2k^3)}=\frac{2^{a+1}}{3a+2}\cdot \frac{3^b}{3b+1}\cdot \frac{5^c}{3c+1}\cdot \frac{d}{\tau(d^3)}\geq \frac{4}{5}\cdot \frac{3}{4}\cdot 1\cdot \frac{7}{4}=\frac{21}{20}>1\]which is an obvious contradiction. Therefore, $k$ is not divisible by any prime number greater than $5.$ Let $k=2^a\cdot 3^b\cdot 5^c$ as before.

Then, we have $2^{a+1}\cdot 3^b\cdot 5^c = (3a+2)(3b+1)(3c+1)$ so clearly, $b=0.$ Thus, $2^{a+1}\cdot 5^c=(3a+2)(5c+1).$

Some more simple, crude, boundings will yield the solutions $n=2,128,2000.$
Z K Y
The post below has been deleted. Click to close.
This post has been deleted. Click here to see post.
mihaig
7368 posts
#18
Y by
Bravo tinere
Z K Y
The post below has been deleted. Click to close.
This post has been deleted. Click here to see post.
megarnie
5610 posts
#19
Y by
ISL marabot solve

Since $n=1$ fails, assume $n>1$. Let the prime factorization of $n=2^{3k+1}\cdot p_1^{3a_1}\cdot p_2^{3a_2}\cdots p_m^{3a_m}$, where $p_i$ are distinct primes greater than $2$.

The equation is $2^{k+1} \cdot p_1^{a_1}\cdot p_2^{a_2}\cdots p_m^{a_m}=(3k+2)(3a_1+1)(3a_2+1)\cdots(3a_m+1)$.

Case 1: $k=0$ or $k=2$.
Then $p_1^{a_1}\cdot p_2^{a_2}\cdots p_m^{a_m}=(3a_1+1)(3a_2+1)\cdots (3a_m+1)$.

Note that $a_i\ne 1$ because the LHS is odd. For $a_i\ge 2$, we get $p_i^{a_i}\ge 3^{a_i}>3a_i+1$. This gives a contradiction unless $m=0$. In this case, $n=2^1=\boxed{2}$ or $n=2^7=\boxed{128}$. Both work.

Case 2: $k=1$.
Then we get $4p_1^{a_1}p_2^{a_2}\cdots p_m^{a_m}=5(3a_1+1)(3a_2+1)\cdots(3a_m+1)$. Clearly, it's not divisible by $3$ and is divisible by $5$, so $p_1=5$.

Note that for $i>1$, we have $p_i^{a_i}>5^{a_i}>3a_i+1$ for all positive integers $a_i$. So we have $4\cdot 5^{a_1}\le 5(3a_1+1)$. This implies $a_1=1$ and $m=1$. So $n=16\cdot 5^3=\boxed{2000}$, which works.

Case 3: $k\ge 3$.
We get $2^{k+1}\ge \frac{16}{11}(3k+2)$. Since both sides are not divisible by $3$, we get $p_i^{a_i}>5^{a_i}>3a_i+1$ for all positive integers $a_i$. This is a contradiction as it implies LHS>RHS.
Z K Y
The post below has been deleted. Click to close.
This post has been deleted. Click here to see post.
awesomeming327.
1732 posts
#20
Y by
Sus

$n=\frac{d(n)^3}{4},$ so if $k=\frac{d(n)}{2}$ then $n=2k^3$ where $k$ is an integer. Let \[n=2^{3e_0+1}\cdot 3^{3e_1}\cdot 5^{3e_2}\cdot \ldots\cdot p_n^{3e_n}\]where $p_i$ denotes the $i$th odd prime, $e_i$ are non-negative integers but $e_n>0$, and $p_n$ is the largest odd prime in the prime factorization of $n$.

Now, the original equation gives \[(3e_0+2)(3e_1+1)(3e_2+1)\cdot \ldots\cdot (3e_n+1)=2^{e_0+1}\cdot 3^{e_1}\cdot 5^{e_2}\cdot\ldots\cdot p_n^{e_n}.\]Note that for $p_i\ge 5,$ $p_i^{e_i}>3e_i+1$ for all $e_i\ge 1.$ Thus, $2^{e_0+1}3^{e_1}\le (3e_0+2)(3e_1+0).$ It is not hard to see that the only solutions of this are:

$(e_0,e_1)=(0,0),(0,1),(1,0),(1,1),(2,0),(2,1).$ For each of those cases, with simple bounding, the answer is $2,128,2000$
Z K Y
The post below has been deleted. Click to close.
This post has been deleted. Click here to see post.
asdf334
7585 posts
#21
Y by
The answers are $n=2$ and $n=128$. (Proof is rushed)

First, note that $n=2k^3$ for some integer $k$. Then note that every exponent $e_p$ of every prime $p$ in the prime factorization of $n$ satisfies $e_p\neq 2\pmod 3$ which means that $3\nmid d(n)$ so that $3\nmid n$. Now suppose that
\[k=2^{e_2}\cdot 5^{e_5}\cdot 7^{e_7}\cdots\]Then define $f(n)=\frac{d(n)^3}{n}$ and note that $f(2)=4$. Suppose we start from $1$ and multiply by $p^{v_p(k)}$ until we reach $k$, counting the value of $f(n)$. It is important to note that $f(n)$ can't decrease on every single multiplication. For primes $p\ge 5$ note that multiplying by any power $p^{e_p}$ will decrease the value of $f$, so any increase must come from the power of $2$. Again, simple bounding gives that any increase must occur when $e_2=0$ or $e_2=2$, with both making $f$ unchanged, meaning that we can't use any primes $p\ge 5$. Our only solutions are $n=2$ and $n=128$.

ACTUALLY, if $e_2=1$, we get that we can't have $e_p>0$ for $p\ge 7$, so a finite case check gives $e_5=1$ which implies $n=2000$ is a final solution.
This post has been edited 2 times. Last edited by asdf334, Jul 29, 2022, 3:34 AM
Z K Y
The post below has been deleted. Click to close.
This post has been deleted. Click here to see post.
lrjr24
967 posts
#22
Y by
We claim the answers are $n=2$, $n=128$, or $n=2000$.
Note that $n=2k^3$, so $d(2k^3)=2k$. Let $k=2^{e_1}3^{e_2}5^{e_3} \cdots p_i^{e_i}$. If there is a prime $p \ge 7$ dividing $k$, then $$\frac{4}{5} \cdot \frac{3}{4} \cdot \frac{p}{4} \le \frac{2^{e_1+1}}{3e_1+2}\frac{3^{e_2}}{3e_2+1} \cdots \frac{p_i^{e_i}}{3e_i+1} = 1,$$a contradiction. This means $k=2^a3^b5^c$. Note that we need to have $\frac{2^{a+1}}{3a+2}\frac{3^b}{3b+1}\frac{5^b}{5b+1}=1$. Note that the minimum possible value of this expression is $\frac{3}{5}$. This means if one of the values is over $\frac{5}{3}$ we get a contradiction. We now track the values each fraction can achieve less than $\frac{5}{3}$. We have that $\frac{2^{a+1}}{3a+2}$ can achieve $1,\frac{4}{5},1, \frac{16}{11}$, $\frac{3^b}{3b+1}$ can achieve $1, \frac{3}{4}, \frac{9}{7}$, and $\frac{5^c}{3c+1}$ can achieve $1, \frac{5}{4}$. If $c=0$, then the only way to achieve reciprocating fractions is if they are both $1$, so we get $k=1$ and $k=4$. If $c=1$, the only way to get the product of $\frac{4}{5}$ is through $a=1$ and $b=0$, so $k=10$. This gives all the solutions and proves there is no more so we are done.
Z K Y
The post below has been deleted. Click to close.
This post has been deleted. Click here to see post.
math_comb01
662 posts
#24
Y by
Sketch
Z K Y
The post below has been deleted. Click to close.
This post has been deleted. Click here to see post.
vsamc
3789 posts
#25
Y by
Solution
Z K Y
The post below has been deleted. Click to close.
This post has been deleted. Click here to see post.
Sagnik123Biswas
421 posts
#26
Y by
This problem can be cracked by making a series of observations.

Consider $d(n)^3 = 4n$. This means that at a structural level, $n = 2j^3$ for some positive integer $j$. Thus we know that $d(n)^3 = 4n = 8j^3 \implies d(2j^3) = 8j^3 \implies d(2j^3) = 2j$.

Now we can arbitrarily think of $j = 2^{e_2}3^{e_3}5^{e_5}7^{e_7} \dots$. Then note that $d(2j^3) = d( 2^{3e_2+1}3^{3e_3}5^{3e_5}7^{3e_7} \dots) = (3e_2 + 2)(3e_3 + 1)(3e_5 + 1) \dots \equiv 2 \pmod{3}$. So if $d(2j^3) \equiv 2 \pmod{3}$ for all $j$ it must be true that $2j \equiv 2 \pmod{3} \implies j \equiv 1 \pmod{3}$.

$j = 1$ obeys $d(2) = 2$. Now consider odd $j \geq 7$. Its prime factorization assumes the form $5^{e_5}7^{e_7}11^{e_{11}} \dots$. If we look at $d(2j^3) = 2(3e_5 + 1)(3e_7 + 1)(3e_{11}+1)\dots$. This needs to equal $2j = 2 \times 5^{e_5}7^{e_7}11^{e_{11}} \dots$ implying that $(3e_5 + 1)(3e_7 + 1)(3e_{11}+1)\dots = 5^{e_5}7^{e_7}11^{e_{11}} \dots$. But note that $p^m > 3m + 1$ for $m \geq 1$ and $p \geq 5$. So this would not be possible.

Thus it must hold that if $j > 1$, it is even. Now let us begin considering $v_2(j)$. If $v_2(j) = 1$, then we now that $j = 2 \times 3^{e_3}5^{e_5}7^{e_7} \dots$. Then $d(2j^3) = 5(3e_5 + 1)(3e_7 + 1) \dots = 2j = 4 \times 5^{e_5}7^{e_7} \dots$. This only holds if $e_5 = 1$.

If $v_2(j) = 2$, we see that the only valid option is no other primes dividing $j$ since $d(2j^3) = 8 \times (3e_5 + 1)(3e_7 + 1)(3e_{11} + 1) \dots = 2j = 8 \times 5^{e_5}7^{e_7}11^{e_{11}} $. Since $p^m > 3m + 1$ for $m \geq 1$ and $p \geq 5$, it must hold that $e_5 = e_7 = e_{11} = 0$.

If $v_2(j) = e_2 \geq 3$, then $3e_2 + 2  < 2^{e_2 + 1}$ not allowing equality.

So $j = 1, 4, 10 \implies n = 2j^3 \implies n = 2, 128, 2000$
This post has been edited 1 time. Last edited by Sagnik123Biswas, Jan 16, 2024, 1:30 AM
Reason: Add some clarity
Z K Y
The post below has been deleted. Click to close.
This post has been deleted. Click here to see post.
pie854
243 posts
#27
Y by
Not a very nice problem

Let $p_k$ be the $k$th prime. Let $n=\prod_{i=1}^\infty p_i^{a_i}$, where only a finite number of $a_i$ are nonzero. So we have $$(a_1+1)^3(a_2+1)^3(a_3+1)^3(a_4+1)^3\cdots=2^{a_1+2}3^{a_2}5^{a_3}7^{a_4} \cdots.$$Note that this implies $3\mid a_1+2$, $3\mid a_2$, $3\mid a_3$, et cetera. We now present a lemma.

Lemma: If $k\geq 0$, $3\mid k$ and $p\geq 5$ then $p^k \geq (k+1)^3$ and equality holds when $k=0$.
Proof. Easy. ////

Case 1: $a_2=0$, $a_1=1$
We have $$(a_3+1)^3(a_4+1)^3(a_5+1)^3\cdots =5^{a_3}7^{a_4}11^{a_5}\cdots$$For this to hold we must have $a_3=a_4=a_5=\dots=0$ due to the lemma. Thus it follows that $n=2$.

Case 2: $a_2=0$, $a_1=4$
We have $$5^3(a_3+1)^3(a_4+1)^3 \cdots =2^6 5^{a_3}7^{a_4} \cdots.$$Note that this implies $a_3\geq 3$. Then we have the inequality $$2^6 5^{a_3} \geq 5^3 (a_3+1)^3.$$With this and the lemma it follows that $a_3=3$, $a_4=a_5=\dots=0$. So $n=2^4\cdot 5^3=2000$.

Case 3: $a_2=0$, $a_1\geq 7$
Then we have the inequality $2^{a_1+2}\geq (a_1+1)^3$ with equality when $a_1=7$. And the lemma gives $a_3=a_4=\dots=0$. So $n=2^7=128$.

Case 4: $a_2=3$
We have $$2^6(a_1+1)^3(a_3+1)^3 \cdots=2^{a_1+2}3^3 5^{a_3} 7^{a_4}\cdots \qquad (1)$$so that $a_1\geq 4$.
If $a_1=4$ then we have $$5^3(a_3+1)^3 \cdots=3^3 5^{a_3} 7^{a_4}\cdots \qquad (2)$$So $a_3\geq 3$, but if $a_3=3$ then by mod $2$ there is a contradiction. So $a_3\geq 6$ in which case we must have $$3^3 5^{a_3} > 5^3 (a_3+1)^3.$$This inequality and the lemma imply that $(2)$ cannot hold.
If $a_1=7$ then it's easy to get a contradiction mod $2$ in $(1)$.
So, $a_1\geq 10$. Then we have the inequality $2^{a_1+2}3^3>2^6(a_1+1)^3$. So once again using the lemma it follows that $(1)$ can't hold.

Case 5: $a_2\geq 6$
The inequality $2^{a_1+2}3^{a_2}>(a_1+1)^3(a_2+1)^3$ holds and so, by the lemma, there's no solution in this case.

We've dealt with all possible cases, so we're done.
This post has been edited 1 time. Last edited by pie854, Sep 29, 2024, 9:15 PM
Z K Y
The post below has been deleted. Click to close.
This post has been deleted. Click here to see post.
ezpotd
1287 posts
#28
Y by
I claim the answers are only $2,128,2000$. It is easy to see that all of these work.

Observe the function $\frac{d(n)^3}{n}$ is multiplicative over prime powers. Clearly, $d(n)$ and $n$ are even. Analyzing the exponents of the primes in $n$, we see they are all divisible by $3$ except the power of $2$, which has exponent $1$ mod $3$. By the number of divisors formula, this gives $d(n) \equiv 2 \mod 3$, so $d(n)$ and $n$ both do not divide $3$.

We now consider the power of $2$ in $n$. Note that for all prime powers other than the powers of $2$ up to $2^10$, some of the powers of $3$ (which don't matter, since $n$ can't divide $3$ anyways), and $5, 5^2,7$, we have $\frac{d(n)^3}{n} < 1$. Letting the power of $2$ in $n$ be $2^x$, we see that $\frac{d(2^x)}{2^x} \ge \frac{4}{\frac 85 \frac 87}$, since all the other prime powers decrease the value of the product. This then forces $0 < x < 9$, we also know that $x \equiv 1 mod 3$, so $x$ can be either $1,4,7$. In both the first and third cases, we have $\frac{d(2^x)}{2^x} = 4$, so we require the product of $\frac{d(p)^3}{p}$ for the other prime powers $p$ to be $1$. Clearly, one prime power must be one of $5, 5^2, 7$, since the total product is $1$ and no individual factor is $1$. We see $5,7$ fail because we would require a power of $2$ in the denominator of the product to cancel out the $8$ on the numerator, impossible, likewise $5^2$ fails because we would require a power of $3$ in the denominator. Thus no set of odd prime powers gives the desired product, so we get the desired solutions of $2, 2^7$. In the other case, we have $\frac{d(2^x)}{2^x} = \frac{125}{16}$, so we must have a power of $5$ to cancel out the $125$ on the top, note that the power of $5$ must be a cube, so testing we see that $5^3$ just works (and we cannot add anymore prime powers, since no set of prime powers $p$ has the product of $\frac{d(p)^3}{p} = 1$ ), and any larger powers of $125$ fail due to size reasons.
Z K Y
The post below has been deleted. Click to close.
This post has been deleted. Click here to see post.
Maximilian113
575 posts
#29
Y by
bruh so much cases

Clearly $n=2k^3$ for some $k \in \mathbb Z^+.$ Let $k=2^x \prod p_i^{e_i},$ then it suffices to solve $$2^{x+1} \prod p_i^{e_i} = (3x+2) \prod (3e_i+1).$$If $x \geq 3,$ $$\frac{2^{x+1}}{3x+2} \geq \frac{16}{11}.$$If $k$ has at least two prime factors other than $2$ $$\frac{\prod p_i^{e_i}}{\prod (3e_i+1)} \geq \frac34 \cdot \frac54.$$So then $$1 = \frac{2^{x+1}}{3x+2} \cdot \frac{\prod p_i^{e_i}}{\prod (3e_i+1)} \geq \frac{15}{11},$$contradiction. But if $x \geq 3,$ and $k$ has a prime factor other than $2$ greater than $3$ we get a similar result. But checking when $k$ has a prime factor of $3,$ if the exponent is at least $2$ we get a size contradiction, and if $k$ is $2^x \cdot 3$ the equation becomes $$2^{x+1} \cdot 3 = (3x+2) \cdot 4$$which can be manually checked to have no solution. Finally if $k=1$ we also have a size contradiction as $2^{x+1} > 3x+2.$

Thus $x \leq 2.$ If $x=0, 2$ then $2^{x+1}=3x+2,$ so $$\prod p_i^{e_i} = \prod (3e_i+1).$$By similar reasoning to above $k=1$ which yields the solutions $n=2, 2^7.$ If $x=1,$ we have $$4 \cdot \prod p_i^{e_i} = 5 \cdot \prod (3e_i+1),$$by similar arguments to above we can't have primes at least $7$ and exponent at least $2$ so we check $k=2 \cdot 3, k=2 \cdot 5$ and only $k=10$ ends up working. This yields $n=2000.$

Hence the solutions are $$n=2,128, 2000,$$which all clearly work.
Z K Y
N Quick Reply
G
H
=
a